23.07.2012 Views

Linear Algebra

Linear Algebra

Linear Algebra

SHOW MORE
SHOW LESS

You also want an ePaper? Increase the reach of your titles

YUMPU automatically turns print PDFs into web optimized ePapers that Google loves.

Topic: Analyzing Networks 77<br />

We can use Kirchhoff’s Law, that the flow into any intersection equals the flow<br />

out, to establish some equations modeling how traffic flows work here.<br />

(a) Label each of the three arcs of road in the circle with a variable. For each of<br />

the three in-out intersections, get an equation describing the traffic flow at that<br />

node.<br />

(b) Solve that system.<br />

3 This is a map of a network of streets. Below we will describe the flow of cars<br />

into, and out of, this network.<br />

Willow<br />

Shelburne St<br />

Jay Ln<br />

west Winooski Ave<br />

The hourly flow of cars into this network’s entrances, and out of its exits can be<br />

observed.<br />

east Winooski west Winooski Willow Jay Shelburne<br />

into 100 150 25 – 200<br />

out of 125 150 50 25 125<br />

(The total in must approximately equal the total out over a long period of time.)<br />

Once inside the network, the traffic may proceed in different ways, perhaps<br />

filling Willow and leaving Jay mostly empty, or perhaps flowing in some other<br />

way. We can use Kirchhoff’s Law that the flow into any intersection equals the<br />

flow out.<br />

(a) Determine the restrictions on the flow inside this network of streets by setting<br />

up a variable for each block, establishing the equations, and solving them. Notice<br />

that some streets are one-way only. (Hint: this will not yield a unique solution,<br />

since traffic can flow through this network in various ways. You should get at<br />

least one free variable.)<br />

(b) Suppose some construction is proposed for Winooski Avenue East between<br />

Willow and Jay, so traffic on that block will be reduced. What is the least<br />

amount of traffic flow that can be allowed on that block without disrupting the<br />

hourly flow into and out of the network?<br />

4 Calculate the amperages in this network with more than one voltage rise.<br />

1.5 volt<br />

5ohm<br />

2ohm 3volt<br />

10 ohm<br />

east<br />

3ohm<br />

6ohm<br />

5 In the circuit with the 8 ohm and 12 ohm resistors in parallel, the electric current<br />

away from and back to the battery was found to be 25/6 amperes. Thus, the

Hooray! Your file is uploaded and ready to be published.

Saved successfully!

Ooh no, something went wrong!